Last visit was: 18 Nov 2025, 19:58 It is currently 18 Nov 2025, 19:58
Close
GMAT Club Daily Prep
Thank you for using the timer - this advanced tool can estimate your performance and suggest more practice questions. We have subscribed you to Daily Prep Questions via email.

Customized
for You

we will pick new questions that match your level based on your Timer History

Track
Your Progress

every week, we’ll send you an estimated GMAT score based on your performance

Practice
Pays

we will pick new questions that match your level based on your Timer History
Not interested in getting valuable practice questions and articles delivered to your email? No problem, unsubscribe here.
Close
Request Expert Reply
Confirm Cancel
705-805 Level|   Weaken|         
User avatar
Bunuel
User avatar
Math Expert
Joined: 02 Sep 2009
Last visit: 18 Nov 2025
Posts: 105,355
Own Kudos:
778,086
 [4]
Given Kudos: 99,964
Products:
Expert
Expert reply
Active GMAT Club Expert! Tag them with @ followed by their username for a faster response.
Posts: 105,355
Kudos: 778,086
 [4]
Kudos
Add Kudos
4
Bookmarks
Bookmark this Post
Most Helpful Reply
User avatar
Bunuel
User avatar
Math Expert
Joined: 02 Sep 2009
Last visit: 18 Nov 2025
Posts: 105,355
Own Kudos:
778,086
 [1]
Given Kudos: 99,964
Products:
Expert
Expert reply
Active GMAT Club Expert! Tag them with @ followed by their username for a faster response.
Posts: 105,355
Kudos: 778,086
 [1]
1
Kudos
Add Kudos
Bookmarks
Bookmark this Post
General Discussion
User avatar
Cana1766
Joined: 26 May 2024
Last visit: 15 Nov 2025
Posts: 85
Own Kudos:
79
 [1]
Given Kudos: 11
Posts: 85
Kudos: 79
 [1]
1
Kudos
Add Kudos
Bookmarks
Bookmark this Post
User avatar
Harika2024
Joined: 27 Jul 2024
Last visit: 18 Nov 2025
Posts: 80
Own Kudos:
65
 [1]
Given Kudos: 31
Location: India
Posts: 80
Kudos: 65
 [1]
1
Kudos
Add Kudos
Bookmarks
Bookmark this Post
Let's check the given options:

(A) Assumes proving many falls have another cause shows falls never indicate neurological decline.
This directly hits the core flaw. The researcher found an alternative cause for most cases in a specific group, but then generalizes this to mean that falls never indicate neurological decline, even in the remaining 38% of that group, or in other groups not studied, or when neurological decline might be a contributing factor even if not the primary one. This is a leap from "sometimes another cause" to "never neurological decline."

(B) Ignores that stiff joints and early neurological decline could coexist in some patients.
This is a plausible criticism. If neurological decline can also be present in some of the 62% who have joint stiffness, then the conclusion is weakened. The researcher states "not because of neurological decline," implying an exclusive cause. If coexistence is possible, then joint stiffness doesn't entirely rule out neurological decline as a contributing or co-occurring factor. This is a strong contender.

(C) Takes for granted that “three or more falls” captures every clinically meaningful fall pattern.
This is a valid point of criticism for a study design, but it doesn't directly address the leap in logic from the premise to the conclusion about neurological decline. The researcher defined their group (3+ falls), and the conclusion is drawn from that group. The vulnerability isn't primarily about the definition of "frequent falls" but about the generalization of the findings.

(D) Presumes the 62 percent figure is inherently large enough to overturn the diagnostic link.
This is also a strong contender. The researcher sees 62% and concludes "therefore, clinicians should no longer regard..." This implies 62% is sufficient to dismiss the previous diagnostic link entirely. Is 62% enough to say never? This is similar to (A).

(E) Overlooks that participants may have been selected precisely because of musculoskeletal problems.
This addresses potential selection bias in the study. If the study participants were pre-selected due to their musculoskeletal issues, then the finding that their falls are due to musculoskeletal issues is less surprising and less generalizable. This would undermine the strength of the evidence for the premise, but it doesn't directly point to the flaw in the reasoning from the premise to the conclusion as strongly as (A) or (B).

on Comparing both (A) and (B), we can see (A) is the most encompassing criticism of the argument's reasoning. It directly challenges the conclusion's universality based on a partial finding.
User avatar
simondahlfors
Joined: 24 Jun 2025
Last visit: 23 Sep 2025
Posts: 48
Own Kudos:
46
 [1]
Posts: 48
Kudos: 46
 [1]
1
Kudos
Add Kudos
Bookmarks
Bookmark this Post
Information given:
- Among adults over 75, 62% of those who suffered three or more falls in the last 12 months walked unsteadily because of joint stiffness and muscle loss, not because of neurological decline
- Therefore, the medical researcher concludes that clinicians should no longer regard a history of frequent falls as evidence that neurological degeneration is beginning

Question:
- The researcher's argument is most vulnerable to criticism on which of the following grounds?

Solution:
- A: Assumes proving many falls have another cause shows falls never indicate neurological decline.
- Just because many falls have other causes, does not mean no falls are linked to neurological decline
- This is a key unwarranted assumption
- Valid

- B: Ignores that stiff joints and early neurological decline could coexist in some patients
- Points out that the argument overlooks that people can have both musculoskeletal problems and neurological degeneration at the same time
- But falls to get the full scope of the flaw (unlike A)
- Invalid

- C: Takes for granted that “three or more falls” captures every clinically meaningful fall pattern.
- The argument doesn't depend on the threshold being all-inclusive
- Invalid

- D: Presumes the 62 percent figure is inherently large enough to overturn the diagnostic link.
- The problem isn't the size of the number, but rather the assumption that having a common alternative cause means falls are no longer diagnostic at all
- Invalid

- E: Overlooks that participants may have been selected precisely because of musculoskeletal problems.
- Suggests a sampling flaw, but the core vulnerability is the assumption in A
- Invalid

Answer: A, Assumes proving many falls have another cause shows falls never indicate neurological decline.
Bunuel
Medical Researcher: Among adults over 75, 62 percent of those who suffered three or more falls in the last 12 months walked un­steadily because of joint stiffness and age-related muscle loss, not because of neurological decline. Therefore, clinicians should no longer regard a history of frequent falls as evidence that neurological degeneration is beginning.

The researcher's argument is most vulnerable to criticism on which of the following grounds?

(A) Assumes proving many falls have another cause shows falls never indicate neurological decline.
(B) Ignores that stiff joints and early neurological decline could coexist in some patients.
(C) Takes for granted that “three or more falls” captures every clinically meaningful fall pattern.
(D) Presumes the 62 percent figure is inherently large enough to overturn the diagnostic link.
(E) Overlooks that participants may have been selected precisely because of musculoskeletal problems.



 


This question was provided by GMAT Club
for the GMAT Club Olympics Competition

Win over $30,000 in prizes such as Courses, Tests, Private Tutoring, and more

 

User avatar
Jarvis07
Joined: 06 Sep 2017
Last visit: 18 Nov 2025
Posts: 295
Own Kudos:
236
 [1]
Given Kudos: 160
GMAT 1: 750 Q50 V41
GMAT 1: 750 Q50 V41
Posts: 295
Kudos: 236
 [1]
1
Kudos
Add Kudos
Bookmarks
Bookmark this Post
The researcher sees that most seniors who fall three or more times do so because of stiff joints or muscle weakness and then concludes that frequent falls can never signal early neurological problems. That’s like saying “Because most apples in the basket are red, none of them can ever be green”, it ignores the fact that even if a majority of falls have a non‐neurological cause, some falls might still reflect the start of neurological decline. This is exactly the overreach described in choice (A).
The other choices miss the heart of the mistake. It isn’t about stiff joints and neurological issues co‐occurring (B), or about whether “three or more” is the right cutoff for a troublesome fall pattern (C), or simply about whether 62 percent is a big enough number to change how doctors think (D), nor does it rest on any hidden bias in how patients were chosen (E). The real problem is the blanket rule that “because most repeat falls aren’t neurological, none ever are,” which is why (A) is the best criticism.

Bunuel
Medical Researcher: Among adults over 75, 62 percent of those who suffered three or more falls in the last 12 months walked un­steadily because of joint stiffness and age-related muscle loss, not because of neurological decline. Therefore, clinicians should no longer regard a history of frequent falls as evidence that neurological degeneration is beginning.

The researcher's argument is most vulnerable to criticism on which of the following grounds?

(A) Assumes proving many falls have another cause shows falls never indicate neurological decline.
(B) Ignores that stiff joints and early neurological decline could coexist in some patients.
(C) Takes for granted that “three or more falls” captures every clinically meaningful fall pattern.
(D) Presumes the 62 percent figure is inherently large enough to overturn the diagnostic link.
(E) Overlooks that participants may have been selected precisely because of musculoskeletal problems.



 


This question was provided by GMAT Club
for the GMAT Club Olympics Competition

Win over $30,000 in prizes such as Courses, Tests, Private Tutoring, and more

 

User avatar
Heix
Joined: 21 Feb 2024
Last visit: 18 Nov 2025
Posts: 361
Own Kudos:
153
 [1]
Given Kudos: 63
Location: India
Concentration: Finance, Entrepreneurship
GMAT Focus 1: 485 Q76 V74 DI77
GPA: 3.4
WE:Accounting (Finance)
Products:
GMAT Focus 1: 485 Q76 V74 DI77
Posts: 361
Kudos: 153
 [1]
1
Kudos
Add Kudos
Bookmarks
Bookmark this Post
Premise: 62% of adults over 75 who had 3+ falls in the last year walked unsteadily because of joint stiffness and muscle loss, not neurological decline.
Conclusion: Clinicians should no longer regard frequent falls as evidence of beginning neurological degeneration.
Analysis: The researcher makes a leap from finding that most falls (62%) are caused by physical issues to concluding that falls should no longer be considered evidence of neurological problems at all. This is an overgeneralization.
Let's evaluate each answer choice:
(A) Correct. This accurately identifies the flaw. The researcher assumes that because many falls (62%) have another cause (joint stiffness/muscle loss), falls should never be used as evidence of neurological decline. This is faulty reasoning - even if most falls have a non-neurological cause, falls could still be a valid indicator of neurological problems in some patients.
(B) Incorrect. While this could be true, the argument doesn't necessarily ignore the possibility of coexistence. The researcher specifically states that the 62% walked unsteadily "not because of neurological decline, suggesting they've accounted for this possibility.
(C) Incorrect. The argument doesn't depend on "three or more falls" capturing every meaningful fall patter. The conclusion is about frequent falls generally.
(D) Incorrect. The issue isn't whether 62% is large enough; it's the leap from "most falls have another cause" to "falls should never indicate neurological decline."
(E) Incorrect. There's no information suggesting selection bias in how participants were chosen.
User avatar
Ryga
Joined: 12 Aug 2023
Last visit: 19 Aug 2025
Posts: 68
Own Kudos:
51
 [1]
Given Kudos: 5
Location: India
Concentration: General Management, Leadership
GMAT Focus 1: 695 Q90 V80 DI83
GMAT Focus 1: 695 Q90 V80 DI83
Posts: 68
Kudos: 51
 [1]
1
Kudos
Add Kudos
Bookmarks
Bookmark this Post
A : The researcher uses the fact that a majority (62%) of cases are not neurological to dismiss the idea that any falls should suggest neurological problems.
This is a classic overgeneralization — just because many fallers have non-neuro issues doesn’t mean none do.

(B): Ignores that stiff joints and early neurological decline could coexist.
(C): Takes for granted that “three or more falls” captures every clinically meaningful fall pattern.
(D): Presumes the 62% figure is inherently large enough to overturn the diagnostic link.
(E): Overlooks that participants may have been selected precisely because of musculoskeletal problems.

Answer A
User avatar
Emkicheru
Joined: 12 Sep 2023
Last visit: 12 Sep 2025
Posts: 119
Own Kudos:
Given Kudos: 11
Location: Kenya
GMAT 1: 780 Q50 V48
GRE 1: Q167 V164
GPA: 3.7
GMAT 1: 780 Q50 V48
GRE 1: Q167 V164
Posts: 119
Kudos: 22
Kudos
Add Kudos
Bookmarks
Bookmark this Post
Bunuel
Medical Researcher: Among adults over 75, 62 percent of those who suffered three or more falls in the last 12 months walked un­steadily because of joint stiffness and age-related muscle loss, not because of neurological decline. Therefore, clinicians should no longer regard a history of frequent falls as evidence that neurological degeneration is beginning.

The researcher's argument is most vulnerable to criticism on which of the following grounds?

(A) Assumes proving many falls have another cause shows falls never indicate neurological decline.
(B) Ignores that stiff joints and early neurological decline could coexist in some patients.
(C) Takes for granted that “three or more falls” captures every clinically meaningful fall pattern.
(D) Presumes the 62 percent figure is inherently large enough to overturn the diagnostic link.
(E) Overlooks that participants may have been selected precisely because of musculoskeletal problems.



 


This question was provided by GMAT Club
for the GMAT Club Olympics Competition

Win over $30,000 in prizes such as Courses, Tests, Private Tutoring, and more

 

Option E can be used as a criticism simply because the researcher did not take into account muscular skeleton problems while making his conclusions.
User avatar
A_Nishith
Joined: 29 Aug 2023
Last visit: 12 Nov 2025
Posts: 455
Own Kudos:
199
 [1]
Given Kudos: 16
Posts: 455
Kudos: 199
 [1]
1
Kudos
Add Kudos
Bookmarks
Bookmark this Post
The medical researcher argues that because a majority (62%) of frequent fallers over 75 had joint stiffness and muscle loss (not neurological decline), frequent falls should no longer be considered evidence of neurological degeneration.

Let's analyze the weaknesses in this argument:

The argument focuses on some frequent falls having another cause, and then makes a sweeping conclusion that falls never indicate neurological decline. The 62% figure leaves 38% of frequent fallers whose unsteadiness could be related to neurological decline, or another cause not mentioned. Even if 62% of frequent fallers are due to musculoskeletal issues, it doesn't mean the remaining 38% are not due to neurological issues, nor does it mean that falls never indicate neurological decline. The conclusion jumps from "many are not due to neurological decline" to "no longer regard... as evidence that neurological degeneration is beginning."

Let's evaluate the options:

(A) Assumes proving many falls have another cause shows falls never indicate neurological decline. This perfectly captures the flaw. The researcher identifies a different cause for a majority of falls and then jumps to the conclusion that falls never indicate neurological decline. This is an overgeneralization. Even if 62% are due to musculoskeletal issues, the remaining 38% could still be due to neurological decline, or falls could indicate neurological decline in other cases not captured by this specific group.

(B) Ignores that stiff joints and early neurological decline could coexist in some patients. While true, this doesn't directly attack the researcher's conclusion about evidence. The researcher's claim is about what should no longer be considered evidence. Even if they coexist, if the cause of the unsteadiness in the 62% group was purely musculoskeletal, the researcher's observation about that group stands, but the conclusion is still too broad. The core flaw isn't that they coexist, but the leap that no falls ever indicate neurological decline.

(C) Takes for granted that “three or more falls” captures every clinically meaningful fall pattern. This points to a potential limitation of the data (what if some other "fall pattern" does indicate neurological decline?), but it doesn't directly target the logical leap the researcher makes based on the 62% figure. The researcher's conclusion is about whether any history of frequent falls should be regarded as such evidence, given their finding.

(D) Presumes the 62 percent figure is inherently large enough to overturn the diagnostic link. This is a strong contender. The researcher is using the 62% as justification for their sweeping conclusion. The argument is vulnerable if 62% isn't considered "large enough" to justify saying all history of frequent falls is no longer evidence. This is very similar to (A) but focuses on the sufficiency of the percentage. However, (A) is broader and more fundamental to the logical fallacy. Even if 62% is "large," it doesn't mean the remaining falls aren't indicative of neurological decline. The problem is the word "never" or "no longer regard...as evidence" when there's still a significant unaddressed portion.

(E) Overlooks that participants may have been selected precisely because of musculoskeletal problems. This points to a potential sampling bias. If the study intentionally focused on people with musculoskeletal problems, the 62% figure might not be representative of all adults over 75 who suffer frequent falls. This would indeed weaken the argument by undermining the representativeness of the data.

Answer: A
User avatar
Rakshit25
Joined: 16 Jun 2020
Last visit: 18 Nov 2025
Posts: 74
Own Kudos:
34
 [1]
Given Kudos: 25
Products:
Posts: 74
Kudos: 34
 [1]
1
Kudos
Add Kudos
Bookmarks
Bookmark this Post
Going to go with A

Just because 60 odd percent falls had a different cause, it doesn't make sense to complete dismiss falls as a cause of neurological decline. It makes sense to say that maybe the probability is less, but not considering it completely is a major flaw.

Option D was also a strong contender for me, but dismissed it since the author doesn't presume the strength of the 62% statistic. It is an observation and no commentary is made on how strong of an observation it is
User avatar
D3N0
Joined: 21 Jan 2015
Last visit: 12 Nov 2025
Posts: 587
Own Kudos:
572
 [1]
Given Kudos: 132
Location: India
Concentration: Operations, Technology
GMAT 1: 620 Q48 V28
GMAT 2: 690 Q49 V35
WE:Operations (Retail: E-commerce)
Products:
GMAT 2: 690 Q49 V35
Posts: 587
Kudos: 572
 [1]
1
Kudos
Add Kudos
Bookmarks
Bookmark this Post
Ans: A

Medical Researcher: Among adults over 75, 62 percent of those who suffered three or more falls in the last 12 months walked un­steadily because of joint stiffness and age-related muscle loss, not because of neurological decline. Therefore, clinicians should no longer regard a history of frequent falls as evidence that neurological degeneration is beginning.

The researcher's argument is most vulnerable to criticism on which of the following grounds?

(A) Assumes proving many falls have another cause shows falls never indicate neurological decline.
What if, the falls were the result of the neurological decline? In this case, argument is vulnerable to criticism.

(B) Ignores that stiff joints and early neurological decline could coexist in some patients.
The argument of the MR does not discuss the idea of co-existing stiff joints and neurological decline together.

(C) Takes for granted that “three or more falls” captures every clinically meaningful fall pattern.
I am not sure about this option as the relationship between the number of falls or any pattern and neurological decline is not established here.

(D) Presumes the 62 percent figure is inherently large enough to overturn the diagnostic link.
significance of 62% as a figure is not part of the argument in the MR's conclusion

(E) Overlooks that participants may have been selected precisely because of musculoskeletal problems.
musculoskeletal problems and any relationship with neuro* declien is not established in the option.
User avatar
Archit3110
User avatar
Major Poster
Joined: 18 Aug 2017
Last visit: 18 Nov 2025
Posts: 8,423
Own Kudos:
Given Kudos: 243
Status:You learn more from failure than from success.
Location: India
Concentration: Sustainability, Marketing
GMAT Focus 1: 545 Q79 V79 DI73
GMAT Focus 2: 645 Q83 V82 DI81
GPA: 4
WE:Marketing (Energy)
GMAT Focus 2: 645 Q83 V82 DI81
Posts: 8,423
Kudos: 4,979
Kudos
Add Kudos
Bookmarks
Bookmark this Post
argument :
Medical Researcher: Among adults over 75, 62 percent of those who suffered three or more falls in the last 12 months walked un­steadily because of joint stiffness and age-related muscle loss, not because of neurological decline. Therefore, clinicians should no longer regard a history of frequent falls as evidence that neurological degeneration is beginning.

weaken

(A) Assumes proving many falls have another cause shows falls never indicate neurological decline. argument is not considering many that falls is not possible because of neurological decline..
(B) Ignores that stiff joints and early neurological decline could coexist in some patients. true and valid to weaken the argument
(C) Takes for granted that “three or more falls” captures every clinically meaningful fall pattern. not all fall are known and reason what led to ..
(D) Presumes the 62 percent figure is inherently large enough to overturn the diagnostic link. does not strengthen the case here..
(E) Overlooks that participants may have been selected precisely because of musculoskeletal problems. irrelevant to argument

OPTION B is correct

Bunuel
Medical Researcher: Among adults over 75, 62 percent of those who suffered three or more falls in the last 12 months walked un­steadily because of joint stiffness and age-related muscle loss, not because of neurological decline. Therefore, clinicians should no longer regard a history of frequent falls as evidence that neurological degeneration is beginning.

The researcher's argument is most vulnerable to criticism on which of the following grounds?

(A) Assumes proving many falls have another cause shows falls never indicate neurological decline.
(B) Ignores that stiff joints and early neurological decline could coexist in some patients.
(C) Takes for granted that “three or more falls” captures every clinically meaningful fall pattern.
(D) Presumes the 62 percent figure is inherently large enough to overturn the diagnostic link.
(E) Overlooks that participants may have been selected precisely because of musculoskeletal problems.



 


This question was provided by GMAT Club
for the GMAT Club Olympics Competition

Win over $30,000 in prizes such as Courses, Tests, Private Tutoring, and more

 

User avatar
Mohak01
Joined: 05 Sep 2020
Last visit: 12 Nov 2025
Posts: 104
Own Kudos:
Given Kudos: 70
Location: India
GMAT Focus 1: 695 Q83 V87 DI83
GPA: 8
GMAT Focus 1: 695 Q83 V87 DI83
Posts: 104
Kudos: 64
Kudos
Add Kudos
Bookmarks
Bookmark this Post
For an easier interpretation, consider a set of 100 patients. Then no. of patients with frequent fall,
due to joints issue - J - 62%
due to neuroglia - N>38%

Consider if patients with both neuroglia and joints issue >24%, then patients with total neuroglia would be greater than 62% and then early signs of neuroglia can't be ignores.

This is targeted by option B.
User avatar
hakzarif
Joined: 31 May 2025
Last visit: 25 Oct 2025
Posts: 65
Own Kudos:
Given Kudos: 9
Products:
Posts: 65
Kudos: 29
Kudos
Add Kudos
Bookmarks
Bookmark this Post
The researcher's argument is flawed because it assumes that frequent falls caused by joint stiffness and muscle loss exclude the possibility of neurological decline, ignoring that these conditions could coexist in some patients. Option (A) points out an overgeneralization, but the main issue is the failure to consider overlapping causes, which is more directly addressed in (B). Option (C) concerns measurement but doesn’t undermine the causal claim, while (D) questions the sufficiency of the 62 percent figure without clear basis. Option (E) suggests possible sampling bias but is speculative. Therefore, the argument’s greatest vulnerability is that it ignores the possibility that stiff joints and early neurological problems can occur together, making (B) the strongest criticism.
avatar
ManifestDreamMBA
Joined: 17 Sep 2024
Last visit: 18 Nov 2025
Posts: 1,282
Own Kudos:
784
 [1]
Given Kudos: 236
Products:
Posts: 1,282
Kudos: 784
 [1]
1
Kudos
Add Kudos
Bookmarks
Bookmark this Post
Medical Researcher: Among adults over 75, 62 percent of those who suffered three or more falls in the last 12 months walked un­steadily because of joint stiffness and age-related muscle loss, not because of neurological decline. Therefore, clinicians should no longer regard a history of frequent falls as evidence that neurological degeneration is beginning.

Prethinking:
- This study is for adults over 75. What if neurological degeneration falls happened or started earlier?
- 62 percent of those who suffered three or more falls in the last 12 months walked un­steadily because of joint stiffness and not because of neurological decline. What if falls before the last 12 months were primarily because of neurological decline or the neurological decline led to joint stiffness?

Checking options...

(A) Assumes proving many falls have another cause shows falls never indicate neurological decline. This is aligned with out second pre-thinking point. it assumes that the stiff joint related falls do not indicate neurological decline
(B) Ignores that stiff joints and early neurological decline could coexist in some patients. The passage is more focused on causation than the correlation.
(C) Takes for granted that “three or more falls” captures every clinically meaningful fall pattern. This isn't suggested in the passage. It is one of the many characteristics of the falls/subjects
(D) Presumes the 62 percent figure is inherently large enough to overturn the diagnostic link. This isn't suggested in the passage. It is one of the many characteristics of the falls/subjects
(E) Overlooks that participants may have been selected precisely because of musculoskeletal problems. This is irrelevant
Bunuel
Medical Researcher: Among adults over 75, 62 percent of those who suffered three or more falls in the last 12 months walked un­steadily because of joint stiffness and age-related muscle loss, not because of neurological decline. Therefore, clinicians should no longer regard a history of frequent falls as evidence that neurological degeneration is beginning.

The researcher's argument is most vulnerable to criticism on which of the following grounds?

(A) Assumes proving many falls have another cause shows falls never indicate neurological decline.
(B) Ignores that stiff joints and early neurological decline could coexist in some patients.
(C) Takes for granted that “three or more falls” captures every clinically meaningful fall pattern.
(D) Presumes the 62 percent figure is inherently large enough to overturn the diagnostic link.
(E) Overlooks that participants may have been selected precisely because of musculoskeletal problems.



 


This question was provided by GMAT Club
for the GMAT Club Olympics Competition

Win over $30,000 in prizes such as Courses, Tests, Private Tutoring, and more

 

User avatar
iCheetaah
Joined: 13 Nov 2021
Last visit: 17 Nov 2025
Posts: 81
Own Kudos:
Given Kudos: 1
Location: India
Posts: 81
Kudos: 72
Kudos
Add Kudos
Bookmarks
Bookmark this Post
Medical Researcher: Among adults over 75, 62 percent of those who suffered three or more falls in the last 12 months walked un­steadily because of joint stiffness and age-related muscle loss, not because of neurological decline. Therefore, clinicians should no longer regard a history of frequent falls as evidence that neurological degeneration is beginning.

We are asked to weaken the argument, so let's see the answer choices:

(A) Assumes proving many falls have another cause shows falls never indicate neurological decline.
  • Too extreme. The word "never" is not sitting well here. The argument of the researcher is that since the majority of the people who had >=3 falls have fallen because of reasons other than neurological decline, clinicians should not consider falling as evidence for neurological degeneration. He never suggests that falls are never caused by neurological decline. Eliminate
(B) Ignores that stiff joints and early neurological decline could coexist in some patients.
  • This kinda works. The researcher says that since the majority those who have fallen is due to reason X (joint stiffness), we should that there is Y (neurological decline). What he misses is the fact that X and Y could co-exist, and that's possible for even the remaining 38% people who have fallen >=3 times. Keep
(C) Takes for granted that “three or more falls” captures every clinically meaningful fall pattern.
  • Defining what a clinically meaningful fall pattern is not relevant to the argument. Eliminate.
(D) Presumes the 62 percent figure is inherently large enough to overturn the diagnostic link.
  • This weakens the argument a tad. But it doesn't address the fact as to why falling should not be considered as evidence for neurological decline. Eliminate
(E) Overlooks that participants may have been selected precisely because of musculoskeletal problems.
  • Selection bias holds no water here. Since, we are already given that we are looking at a particular set of adults who are above 75 and within that, those that had >=3 falls in the last 12 months. Eliminate

B seems to be the best choice.
User avatar
Missinga
Joined: 20 Jan 2025
Last visit: 16 Nov 2025
Posts: 393
Own Kudos:
Given Kudos: 29
Posts: 393
Kudos: 261
Kudos
Add Kudos
Bookmarks
Bookmark this Post
Medical Researcher: Among adults over 75, 62 percent of those who suffered three or more falls in the last 12 months walked un­steadily because of joint stiffness and age-related muscle loss, not because of neurological decline. Therefore, clinicians should no longer regard a history of frequent falls as evidence that neurological degeneration is beginning.

The researcher's argument is most vulnerable to criticism on which of the following grounds?

(A) Assumes proving many falls have another cause shows falls never indicate neurological decline....Could be the answer ( it indicates an overgeneralisation)
(B) Ignores that stiff joints and early neurological decline could coexist in some patients.....it seems like a general statement, it does mot weaken the conclusion .....No
(C) Takes for granted that “three or more falls” captures every clinically meaningful fall pattern......No (the argument is about cause of falls not how many falls)
(D) Presumes the 62 percent figure is inherently large enough to overturn the diagnostic link.......No
(E) Overlooks that participants may have been selected precisely because of musculoskeletal problems..... If the sample is biased, then conclusion is wrong .......Possible answer

E
User avatar
iamchinu97
Joined: 14 Dec 2020
Last visit: 18 Nov 2025
Posts: 132
Own Kudos:
139
 [1]
Given Kudos: 34
Products:
Posts: 132
Kudos: 139
 [1]
1
Kudos
Add Kudos
Bookmarks
Bookmark this Post
Medical Researcher: Among adults over 75, 62 percent of those who suffered three or more falls in the last 12 months walked un­steadily because of joint stiffness and age-related muscle loss, not because of neurological decline. Therefore, clinicians should no longer regard a history of frequent falls as evidence that neurological degeneration is beginning.

The researcher's argument is most vulnerable to criticism on which of the following grounds?
=> Here author is trying to prove that A is the only cause of B. and this indicate that C was not the cause. from one observation he is generalizing for all the falls. so any option which suggests that will be our ans.

(A) Assumes proving many falls have another cause shows falls never indicate neurological decline. =>
This is exactly in our line of logic. author is assuming from 62% falls that we should completely remove neurological decline as the cause. but what if remaining 38% were due to neurological declines then authors reasoning vulnerable.

(B) Ignores that stiff joints and early neurological decline could coexist in some patients. => Even if both the exist then author can still argue that the main cause is the stiff joints and muscle loss so this is not strong argument agains author so not the ans

(C) Takes for granted that “three or more falls” captures every clinically meaningful fall pattern. => Here fall patterns from the fall occurred are not impacting the argument and not attacking reasoning or author so not the ans

(D) Presumes the 62 percent figure is inherently large enough to overturn the diagnostic link. => Even if autor assumes that 62% is large enough to weaken the argument we need to show other cause still exists and can't be completely ignored even when it is in 5% of the patients. so not the ans

(E) Overlooks that participants may have been selected precisely because of musculoskeletal problems. => Okay selection of participants is not in the scope and no info on it is given so not the ans we are not concerned about the participants as we are more focus on whether the neurological cause is there or not.

Hence Ans A
User avatar
jipandeyji
Joined: 10 Jan 2016
Last visit: 16 Nov 2025
Posts: 27
Own Kudos:
Given Kudos: 62
Posts: 27
Kudos: 21
Kudos
Add Kudos
Bookmarks
Bookmark this Post
Answer A -
Just because many cases of falling are due to other causes, it does not follow that no cases are due to neurological decline. The researcher assumes that once a different cause is common, the original cause can be dismissed entirely.
 1   2   3   4   
Moderators:
GMAT Club Verbal Expert
7445 posts
GMAT Club Verbal Expert
234 posts
188 posts